?'s ASWB Assessment

Pataasin ang iyong marka sa homework at exams ngayon gamit ang Quizwiz!

SOAP charting arranges client records according to four components, including: Select one: a. subjective information, objective information, assessments, and prognosis. b. systems evaluation, objectives of treatment, actions, and prognosis. c. subjective information, objective information, actions, and progress. d. subjective information, objective information, assessments, and plan

D: The components of SOAP charting are subjective information about the client (e.g., symptoms reported by the client); objective information about the client (e.g., sociodemographic information); assessments and conclusions based on this information; and a treatment plan for addressing the problem

The parents of a 10-year-old child whom a school social worker has tested ask to look at their child's school record which contains, among other things, the test protocol and the social worker's report. The social worker's policy should be: Select one: a. they cannot see the record, but the social worker can provide them with a summary of its contents. b. they cannot see the record, but they can ask the social worker questions about the test results. c. let them look at the record. d. let them look at some of the material in the record and interpret the material for them so that it doesn't cause them to worry

D: The parents have the right to review the contents of their child's school record, other than the social worker's personal notes and any secure testing material, and the social worker is obligated to interpret the content they review in language they can understand. The Federal Educational Rights and Privacy Act (the Buckley Amendment) applies to educational settings and grants parents and students over the age of 18 the right to inspect their child's or their own educational records

What is the time-frame for achieving steady state on a benzodiazepine? Select one: a. One day to 21 days. b. Approximately two weeks. c. Three to six weeks. d. At least one month

A: When a benzodiazepine is taken daily, the amount of time it takes to achieve "steady state" ranges from one day to about three weeks

An Hispanic individual who recently immigrated to the United States is MOST likely to have difficulty working with a social worker who: Select one: a. adopts an eclectic orientation. b. encourages individualism. c. has an informal personal style. d. displays an interest in the client's level of acculturation

B: The worldview known as collectivism is characteristic of many culturally diverse groups in the United States, including Hispanics. Collectivism assumes that groups connect and mutually obligate individuals - the personal is just one aspect of and is subordinate to the larger social group or context. Individualism, by contrast, centers on the personal - personal goals, personal uniqueness, and personal control - rather than on the social group or social context in which a person lives. Choices "C" and "D" would be advantageous when working with a recent Hispanic immigrant: An informal approach to business relationships is characteristic of Hispanic culture, and an interest in the client's level of acculturation is necessary whenever a therapist is working with a client who is a recent immigrant or refugee. Finally, an eclectic orientation is unlikely to be problematic because it implies greater therapist flexibility

Characteristics of the child that are associated with a high risk for abuse include all of the following, EXCEPT: Select one: a. poor health. b. difficult temperament. c. adolescence. d. disability

C: Characteristics of the child that are associated with a high risk for abuse include prematurity and low birth-weight, difficult temperament (e.g., nonresponsivity, irritability, hyperactivity), poor health, and younger age (children under age 3 are at highest risk for physical abuse). Overall, children who are perceived as "different," such as those with a difficult temperament or a disability, are at increased risk for both abuse and neglect (Goldman et al., 2003)

When using the DSM-5, an individual's functioning in conceptual, social, and practical domains is evaluated to determine the appropriate severity level for which of the following diagnoses? Select one: a. Autism spectrum disorder. b. Specific learning disorder. c. Intellectual disability. d. PTSD

C: DSM-5 distinguishes between four levels of severity for intellectual disability (mild, moderate, severe, and profound) with severity being based on the individual's functioning in conceptual, social, and practical domains. In the DSM-5, the single diagnosis of autism spectrum disorder ("A") encompasses autistic disorder and three other DSM-IV-TR diagnoses - Asperger's disorder, childhood disintegrative disorder, and pervasive developmental disorder NOS. The DSM-5 distinguishes between three levels of severity of autism spectrum disorder: Level 1 (requiring support), Level 2 (requiring substantial support), and Level 3 (requiring very substantial support).

In structural family therapy, what is the purpose of "unbalancing"? Select one: a. To provoke resistance from family members. b. To reframe the problem. c. To disrupt the family's homeostasis. d. To engender insight

C: Structural family therapists view family dysfunction as resulting from an inflexible family structure that prohibits a family from adapting to situational and maturational stressors in a healthy way and believe that improvements in the family process will lead to improvements in individuals within the family. The primary goal of therapy, therefore, is to restructure the family. Restructuring techniques (e.g., reframing, blocking) are used to deliberately unbalance a family's homeostasis (upset the ways in which family members currently relate to one another) in order to bring about a change in the family's structure. By undermining the family's existing homeostasis, the therapist creates a crisis that spurs the system toward the development of a better functioning organization

A number of relapse prevention models have been proposed for alcoholism, cigarette smoking, and other addictive disorders. According to the model developed by Marlatt and his colleagues, what is the MOST effective way to increase the likelihood of recovery after relapse? Select one: a. Continue to promote complete abstinence. b. Emphasize the negative consequences of further "acting out." c. Shift attention from internal to external antecedents. d. Clarify underlying issues

C: The relapse prevention model proposed by Marlatt and his associates is a cognitive-behavioral model that emphasizes the role of cognitions (especially attributions) and the situational factors that precede relapse. Marlatt and Gordon's (1985) "abstinence violation effect" (AVE) model considers recovery after relapse to be related to attributions about the cause of the relapse: Successful recovery from relapse is more likely when the person blames it on external, unstable, and specific (high-risk) factors than when he or she attributes it to internal, stable, and global factors

A client says during the first interview that she regrets coming in because, deep down, she thinks people should be able to solve problems without help. The social worker recognizes this statement as a sign of ambivalence. Which of the following would be the BEST way of responding to what the client has said? Select one: a. "You're right in a way; you are, in the end, the one who has to cope with your own problems. On the other hand, it may help you to cope if you talk about your problems with someone else." b. "As a social worker, I agree with you; in fact, my goal is to empower you to deal with problems on your own." c. "I understand your feelings; however, the fact that you came to me suggests that you are unable to cope on your own right now." d. "I hear what you're saying; this might not be the right time for you to be in therapy, but you're welcome to come back when you're ready."

A: "A" is best because it both acknowledges the client's feelings and addresses her ambivalence by explaining how therapy can provide her with the tools to cope on her own; i.e., it points out that she will play an active role in solving her own problems, even if she accepts your help

In what therapeutic approach does the therapist deliberately stay neutral? Select one: a. Psychoanalytic. b. Client-centered. c. Cognitive. d. Reality

A: Analytic neutrality is a key component of psychoanalysis. The therapist remains objective at all times and doesn't take sides in the client's conflicts; and the therapist's neutrality is believed to allow the client to project onto the therapist feelings he or she originally had for a significant person in the past. Client-centered therapy is based on the premise that, if the "right environment" (i.e., unconditional positive regard, accurate empathic understanding, genuineness) is provided, then the client will achieve congruence between self and experience and be carried by his or her own innate tendency toward self-actualization. In client-centered therapy, the therapist honestly communicates his or her feelings whenever it is appropriate to do so. In some forms of cognitive therapy (e.g., REBT), the therapist may be quite confrontive about the client's cognitions and behaviors. Finally, reality therapy is also confrontive (e.g., it emphasizes value judgments and teaches clients behaviors that will enable them to fulfill their needs)

A client expresses a desire to terminate therapy because she is "feeling much better" and is able to handle her problems successfully. Although this seems appropriate since the client has met her treatment goals, the social worker is reluctant to end therapy and feels like he wants to do more for the client. What should the social worker do? Select one: a. Recognize that his reluctance is normal and assist the client with termination. b. Recognize that his reluctance may indicate that the client is not ready to terminate. c. Refer the client to another therapist because he is no longer objective. d. Seek consultation to address the loss of objectivity before deciding whether the client is ready to terminate

A: Answer "A" is the best choice, partly because you have limited information about this case. What you probably do know is that a social worker's reaction to planned termination may sometimes include a desire to "do more" for a client even though the client has met her treatment goals. This is one reason why, as a social worker, you need to monitor not just the client's reactions to termination, but also your own. If you feel a sense of loss over ending the relationship with a client, you must be careful to continue emphasizing the client's needs over your own

After a single mother is charged with child neglect, her two young children are placed in temporary foster care. The woman eventually meets the goals of her treatment plan, and the court determines that her children can be returned to her care. A few days before the children are scheduled to come home, the woman reveals to her caseworker that she is two months pregnant. What is the caseworker's BEST course of action? Select one: a. Return the children to their mother's care but monitor the case closely. b. Return the children as planned because the woman has met the goals of her treatment plan. c. Make sure that the children remain in foster care. d. Make arrangements to recontact the family after the baby is born

A: Because the woman has met her treatment goals, her children should be returned to her care as "B" suggests. Just doing this is not sufficient, however, because a pregnancy and a third child will produce additional stress for the woman; therefore, the caseworker should also provide structure for the woman by making the return conditional and monitoring the situation. The intervention in answer "D" is insufficient; plus, "D" doesn't indicate whether the caseworker is returning the children to the woman's care at this time

A social worker's new client is a woman who is very concerned because her 18-month-old daughter has not started walking or talking yet. After acknowledging the mother's feelings of concern, what should the social worker do FIRST? Select one: a. Refer the mother to a pediatrician to test the child's vision and hearing. b. Reassure the mother that the child's development is proceeding at a normal rate. c. Refer the child for psychological testing. d. Get thorough developmental and family histories and observe parent-child interactions

A: By age 18 months, a child should be walking and saying at least a few words. A plausible cause of delays in these areas of development is sensory deficits, especially visual and auditory deficits. If sensory deficits are ruled out, then a test such as the Bayley III or Denver II could be used to assess the girl's developmental status or screen for developmental delays. The assessments in "D" also may turn out to be useful, depending on what the physician finds

A social worker has been working in individual therapy with a 19-year-old client who has two younger sisters, ages 15 and 10. The client reports in a session that her father sexually abused her for over 10 years. The social worker knows that the client's sisters live at home with their father. When he asks the client if she believes that her father is molesting her sisters, the client becomes very uncomfortable and quickly changes the subject. She says she doesn't want to talk about her sisters. Regarding the filing of a child abuse report in this situation, what should the social worker do? Select one: a. File a report based on his suspicion that the client's sisters are being abused. b. Report abuse of the client, since abuse of the client began when she was a minor. c. Get more information from the client to determine whether a child abuse report is necessary. d. Maintain the client's confidentiality because she is an adul

A: Child abuse reporting laws generally require mandated reporters (such as social workers) to make a report whenever they, in their professional capacity or within the scope of their employment, have knowledge of or observe a child whom they know or reasonably suspect has been the victim of child abuse or neglect. This client is no longer a minor, and a mandated reporter is not required to make a report when an adult says that she was abused as a child. In this situation, however, the social worker would have very good reason to suspect that the client's sisters, who are minors, are being abused; therefore, he would be mandated to file a child abuse report regarding that (potential) abuse. As a mandated reporter, you don't have to know with certainty that abuse is occurring; a reasonable suspicion is all that is necessary for your mandated reporting obligation to take effect

A social worker is working in individual therapy with a 19-year-old man whose father is paying for his treatment. During a session, the client tells the social worker that he's angry at his father and plans to confront him soon. A few days later, the father calls the social worker demanding to know why she encouraged his son to confront him. What should the social worker do? Select one: a. Refuse to talk with the father about this matter. b. Explain to the father how answering his question would reduce the therapeutic value of her work with his son. c. Apologize to the father and tell him that she will discuss this matter with his son. d. Encourage the father to come in for a family session

A: In the absence of any information about what specific arrangements you have made with the client and his father regarding sharing confidential information with Dad, "A" is the best answer. Parents or others paying for an adult client's treatment sometimes assume that they have a right to be given confidential information about the client; in fact, their involvement does not afford them this right. You should make information available to these entities only at the client's request or when the client has consented to have it released at the request of another, and only when disclosure is in the client's best interests. At the beginning of the therapeutic relationship, you should clarify with all involved persons (in this case, the client and his father) the conditions surrounding the release of information

Loosening of associations can be symptomatic of schizophrenia and other psychotic disorders and involves: Select one: a. abrupt shifts from one topic to an unrelated or obliquely related topic. b. delays in answering a question due to embellishment with unnecessary details and parenthetical remarks. c. a continuous flow of rapid speech with abrupt changes from one topic to another based on understandable associations. d. thinking that contains numerous contradictions and inconsistencies

A: Loose associations are characterized by abrupt shifts from one fragmented thought to another with little if any logical connection between the thoughts. Response "B" defines circumstantiality; "C" refers to flight of ideas; and "D" describes illogical thinking.

Question ID:42380 A client is very quiet in the first interview, and the social worker notices that she client is slouching, often looks down at the floor, and is holding her arms close to her body. According to neurolinguistic programming, what should the social worker do to build trust with this client? Select one: a. Adjust his posture to that of the client. b. Use "feeling" words. c. Use "thinking" words. d. Attempt to make direct eye contact

A: Neurolinguistic programming (NLP) suggests that a therapist's ability to communicate effectively with a client will be influenced by his or her ability to ascertain and then work with the client's preferred sensory mode; e.g. if a client prefers a visual mode, the therapist will use terms such as "I see," if a client prefers an auditory mode, the therapist will use expressions such as "I hear you." "A" is the best choice, because it involves reflecting the kinesthetic state of the client, which is what is emphasized in this question.

A 79-year-old client spends a lot of time in therapy sessions reminiscing about events from her past. What is the social worker's BEST course of action? Select one: a. Be patient and recognize that reminiscing is a healthy activity. b. Encourage the client to find ways of feeling satisfied with her life now rather than dwelling on the past. c. Refocus the client's attention on her present-day problems. d. Explore why the client is focused on the past

A: Older adults often engage in a process of "life review" in which they reminisce, tell stories, and think about past events. This process serves a useful function by helping an older person integrate her life in a meaningful way. When an older adult wishes to talk about her past during sessions, you should incorporate her desire to review her life into your work with her

Social agency administrators are responsible for monitoring the implementation of an agency's policies and for making sure that all agency staff are notified of policy changes. Which of the following word sets BEST captures the key principles associated with effective communication of policy changes to staff? Select one: a. Timely, clear, repetition. b. Timely, verbal communication, memo. c. Immediate, clear, democratic. d. Immediate, descriptive, specific

A: This answer mentions all three elements that contribute to effective communication about policy changes at an agency: Administrators should communicate information about policy changes to the agency's staff in a timely way, in a clear way, and, ideally, both in written form and during staff meetings where staff can ask questions and voice their concerns such as concerns about how the new policy will affect their work. Presenting information both in written form and verbally is important because repetition promotes effective communication among those working at an agency

A 10-year-old boy reports that, lately, he's been smelling unusual odors, like burnt flesh and rotten food, but that none of those things are around him when this happens. He is also having difficulty keeping up in his classes. In assessing this boy, what should the social worker do FIRST? Select one: a. Refer him to a neurologist. b. Refer him to a psychiatrist. c. Arrange to speak to his teacher. d. Refer him to an educational psychologist

A: This boy seems to be experiencing olfactory hallucinations, which could be caused by a neurological disorder (e.g., epilepsy, head injury). The boy's academic problems ("C" and "D") can be dealt with after you know the cause of and appropriate treatment for the hallucinations

During the first therapy session, an Asian American client tells his white social worker that he would prefer to see an Asian American therapist. What is the social worker's BEST course of action? Select one: a. Realize that the client's preference may be a sign of resistance and tell him that she thinks it would be best if they saw each other for a few sessions before a referral is made. b. Discuss this with the client and make a referral if, after the discussion, the client still wants to see an Asian American therapist. c. Tell the client that it would be discriminatory and, therefore, unethical for her to make a referral to another therapist on the basis of race or culture alone. d. Reassure the client that she has experience working with members of minority groups and suggest that they see each on a trial basis

A: This is the best response: The client will not benefit from treatment if he's not comfortable with the therapist, and it would be unethical for the social worker to try to "talk him into" additional sessions

The clients are a family from Mexico who immigrated to the United States 18 months ago. In the first interview, the parents disclose that they recently discovered that their 16-year-old daughter has been sexually active with her 17-year-old boyfriend, who is a classmate of hers. The father then asks the social worker whether she has a daughter of her own. What is the BEST way for the social worker to respond? Select one: a. Answer the question to build rapport. b. Decline to answer the question until after rapport has developed. c. Decline to answer the question to preserve therapeutic boundaries. d. Answer the question only if she is comfortable doing do

A: Usually, when a client asks you a personal question, you can choose to answer it or not answer it depending on what you're uncomfortable doing. And, if you decline to give an answer, you should authentically share your reasons why. In this case, however, there is an additional consideration, other than your own comfort, that should factor into your decision about whether to answer the client's question - i.e., the family's Mexican culture. Given the family's cultural background, your willingness to answer the father's question may be important for facilitating the development of rapport.

A 67-year-old accountant exhibits impairments in memory and other cognitive functions along with a depressed mood. The presence of which of the following suggests that his symptoms are due to pseudodementia (depression) rather than mild neurocognitive disorder? Select one: a. The man tends to exaggerate his cognitive problems. b. The onset of his symptoms was slow and gradual. c. The man's symptoms worsen over the course of the day. d. The man seems to be unaware of his cognitive problems

A: We are looking for the answer that would support a diagnosis of depression for this man. Depression is often difficult to differentiate from mild neurocognitive disorder when the depression includes prominent cognitive symptoms. However, there are some distinguishing characteristics. For example, people with depression are likely to exaggerate their cognitive problems, while those with mild neurocognitive disorder are likely to minimize or deny them. An insidious onset of symptoms ("B") and a worsening of symptoms in the evening ("C") are more characteristic of neurocognitive disorder.

Nonverbal barriers to effective communication with a client include all of the following EXCEPT: Select one: a. leaning forward in your chair. b. sitting at your desk during a session. c. staring at the client to encourage eye contact. d. keeping an even tone as you speak so that your voice does not reveal your reactions to the client

A: You needed to choose the answer that is not a nonverbal barrier to effective communication with a client. Answer "A" describes social worker "attending" behavior which usually facilitates client-social worker communication and rapport. A desk separating you from a client ("B") tends to inhibit openness and conveys that you're in a superior position to the client; and a monotonous or flat voice ("D") communicates a lack of interest

A social worker refers a client with intense social anxiety to a psychiatrist for a medication evaluation and explains to him why she is doing so, including how he might benefit from beginning to take medication. The client becomes defensive when the social worker makes this referral and says, "You think I'm crazy, don't you?" What is the social worker's BEST response? Select one: a. Describe to the client her scope of practice. b. Explore the client's fears or other concerns about the referral. c. Make the initial contact herself by calling the psychiatrist while the client is with her. d. Explain to the client that, ethically, she cannot continue treating him unless he follows through with this referral

B: "B" is much more empathetic than the other answers. Even a client who understands the benefits he stands to gain from using another professional's (or agency's) services may still be fearful or uncertain; when this is the case, you should explore the client's ambivalence about the referral and help him express his fears or other concerns about using the resource (i.e., about taking medication, in this case). Offering a "legalistic" justification for the referral, like the one in "A," would probably mean little to the client, fails to address his underlying concerns, and is likely to increase, rather than allay, his defensiveness and fears. Answer "C" is a connection technique that you may use with some clients, such as those who are anxious or fearful about making the initial contact with a resource themselves. Before deciding what connection techniques to use in this case, you need to address the client's reaction to the referral and see if he will even be willing to meet with the psychiatrist. And, finally, eliminate "D" since there is nothing in NASW's Code of Ethics that would cover this. This response makes it sound as though the client has an obligation to follow all of your recommendations. It also fails to address the client's underlying needs or concerns

A proponent of systems theory might say which of the following about the effects on a family of contact with the environment? Select one: a. The family affects the environment more than the environment affects the family. b. It has a positive effect. c. It has a negative effect. d. It will have no effect if the family has healthy boundaries

B: According to systems theory (or general systems theory), a system with a continuous information flow to and from the outside is an open system, while a system whose boundaries are not easily crossed is a closed system. Thus, an open family system has permeable boundaries that permit a high degree of interaction with and accessibility to its outside environment, while closed family system has impermeable boundaries that permit little interaction with its outside environment. A closed family system, by failing to interact sufficiently with the outside environment, lacks corrective feedback mechanisms, becomes isolated, and resists change. In contrast, an open family system, by welcoming and initiating interactions with the outside environment, is more adaptable and accessible to change; as a result of exchanges beyond its boundaries, an open family system is more able to deal with stress and change and to make necessary and enabling adaptations. Note, however, that no family system is fully open or closed, as a totally open system would have no boundaries and, therefore, would cease to exist as a separate entity; and a totally closed system would have no exchanges at all with its outside environment and would die.

A supervisor of social workers who are involved in community organizing would be MOST likely to focus his efforts on: Select one: a. providing direct supervision. b. dealing with issues of organizational loyalty. c. identifying the needs of the social workers' clients. d. assigning and reviewing the work done by social workers

B: Community organizing relies on a tradition of political activism and a rejection of traditional bureaucratic structures, and community organizers tend to work away from agencies, in the field, where they are subject to pressures and power plays from many different groups. For these reasons, monitoring loyalty to the agency, including accountability to agency administration, is a key task in the supervision of social workers engaged in community organizing: The supervisor needs to make sure that community organizers do not commit the agency to activities or policies it cannot support (Kadushin and Harkness, 2002). At the same time, the more traditional functions of supervision are usually less emphasized because community organizers do their work in a more public setting (e.g., outside of an office). The greater visibility of their work precludes the need for many traditional supervisory processes (e.g., regular conferences to review cases). Moreover, Kadushin and Harkness note that, because community organizers usually work in small agencies with limited staff or are members of small specialized departments within large agencies, supervision in community organizing is generally less explicitly formulated than in most other forms of social work practice, and there may be no hierarchical structure that includes supervisory personnel. An agency administrator, rather than a supervisor, may assign and review the work done by social workers who are community organizers (answer "D")

A social worker is meeting with a mother who gave birth several months ago. The mother drank alcohol regularly while she was pregnant and her baby has been diagnosed with fetal alcohol syndrome. This diagnosis suggests that the baby is MOST likely to exhibit which of the following either now or as he grows older? Select one: a. Intellectual disability, cerebral palsy, growth deficiencies. b. Lower-than-average intelligence, abnormal facial features, growth deficiencies. c. Microencephaly, lower-than-average intelligence, chronic depression. d. Abnormal facial features, delayed motor development, cerebral palsy

B: Fetal alcohol spectrum disorders (FASDs) are a group of conditions that can occur in a child whose mother drank alcohol during pregnancy, and fetal alcohol syndrome (FAS) is the most severe FASD. The symptoms of FAS vary, depending on the amount of alcohol consumed, but may include abnormal facial features, retarded physical growth, heart defects, intellectual disability, hyperactivity, irritability, and central nervous system (CNS) problems. In addition, a person with FAS may have problems with learning, memory, attention span, communication, vision, or hearing. The risk for FAS is highest, and the symptoms are most severe, when the mother drinks heavily every day or, in the early stages of pregnancy, engages in binge drinking. Most symptoms of FAS are irreversible and persist into adulthood

According to Freud's structural theory, cultural values are part of the: Select one: a. ego. b. superego. c. id. d. self-preservation instincts

B: Freud's structural theory posits the personality with three structures - the id, the ego, and the superego. The superego is said to develop from a culmination of maturation, parental identification and modeling, and societal teaching. It represents the social values that are incorporated in the child's personality structure and becomes the conscience that tries to influence behavior so that it conforms to social expectations. Self-preservation instincts ("D") function to preserve biological life. Also known as the life instincts, they include instincts to attain food, water, and air. Freud recognized these basic instincts but placed much greater emphasis on the sexual instinct

A social worker is leading a therapy group made up of female patients who have been ordered into treatment by the court. The patients all have alcohol use disorder. During a group meeting, one of the patients asks the social worker if she's currently dating anyone. How should the social worker respond to this? Select one: a. Redirect the focus by inviting the patients to talk about their relationships. b. Recognize this as an opportunity to build trust and answer the question. c. Decline to answer the question and reinforce the purpose of the group. d. Answer the question to reinforce the norm of self-disclosure

B: Involuntary members can be forced by court order to attend group meetings, but they can't be forced to interact in meaningful and productive ways during group sessions. Effective group work with involuntary clients usually requires a strong group leader who is skilled at confronting resistant and manipulative behavior. "C" would reinforce proper therapeutic boundaries and keep the group members focused on their treatment goals

For Salvador Minuchin, "psychosomatic families" (i.e., those in which asthma, diabetes, or anorexia threaten the life of one child) are MOST likely to be characterized by which of the following? Select one: a. Frequent intense open conflicts between family members. b. Weak boundaries between family members and limited opportunities for individual autonomy. c. Family roles that are inflexible and stereotyped. d. Marked emotional distance between the husband and wife who are both emotionally immature

B: Minuchin is associated with structural family therapy, which emphasizes the nature of the family structure including the boundaries between family members. According to Minuchin, enmeshment, which involves diffuse (weak) boundaries, is common in psychosomatic families. "A" is incorrect because, according to Minuchin, psychosomatic families are characterized by a consistent avoidance of conflict. Ackerman (who was one of the pioneers of family therapy) is more associated with the importance of roles in family dysfunction ("C"); and "D" describes "emotional divorce," which is a term used by Bowen

The clients are a young mother and father who report feeling overwhelmed by caring for their 22-month-old son. They say that he has recently become more difficult to manage than he was before. After acknowledging the parents' feelings, what should the social worker do FIRST? Select one: a. Develop with the parents a list of support people who can be relied on to babysit and offer other support. b. Discuss the developmental stages of early childhood with the parents. c. Help the parents determine whether their son's behavior is within normal developmental limits. d. Assist the parents to identify their frustrations associated with parenting

B: Providing this education would be the best first intervention. The question suggests that these parents lack knowledge about childhood development and may have unrealistic expectations about how children behave at age 22 months (i.e., we would expect a 22-month-old to be more active, mobile, curious, and independent and, therefore, more "difficult to manage than he was before"). Depending on the specific nature of the boy's behaviors (which are not described in the question), "C" may also be indicated but this discussion will be more fruitful after the parents have accurate information about childhood developmental stages.

A social worker determines that a client, age 36, is in the midst of a major depressive episode, and the client's history suggests that he has had other such episodes in the past. The social worker refers the client to a psychiatrist for a medication evaluation, and the psychiatrist prescribes an SSRI for the client. In terms of side-effects from this class of medication, the client is MOST likely to experience which of the following? Select one: a. Blurred vision, tremor, weight loss. b. Gastrointestinal problems, insomnia, sexual dysfunction. c. Dry mouth, tachycardia, weight gain. d. Muscle spasms, confusion, irritability

B: Side-effects associated with the SSRIs (Prozac, Floxyfral, Paxil, Zoloft) include gastrointestinal problems (e.g., appetite loss, nausea, constipation, diarrhea); frequent urination; insomnia; anxiety; headache; dizziness; tremor; and sexual dysfunction. Compared to tricyclic antidepressants (TCAs), SSRIs are safer in overdose; less cardiotoxic; less likely to result in cognitive impairment; and have a more rapid onset. Answer "C" describes side-effects associated with TCAs. These drugs are associated with a variety of anticholinergic side-effects, including dry mouth, sweating, blurred vision, constipation, and urinary retention. TCAs are also cardiotoxic, which means that they may produce cardiovascular side-effects, such as palpitations, tachycardia, hypertension, severe hypotension (drop in blood pressure), and cardiac arrhythmia. Other side-effects may include confusion, disorientation, sedation, rash, paresthesia, blood dyscrasia, nausea, vomiting, weight gain, changes in libido, paradoxical depression or agitation, and lowered seizure threshold

A patient in hospice care tells the social worker that he has decided to end his life so that he can prevent his loved ones from suffering anymore. He describes a fairly detailed suicide plan. What should the social worker do? a. Note this in the patient's record and monitor him closely. b. Inform the hospice medical staff. c. Respect the client's decision because he is terminally ill. d. Offer the client incentives to keep living

B: Some terminally ill people choose to commit suicide as a way of avoiding physical pain and deterioration, dying with dignity, and or or sparing their loved ones from experiencing a lengthy dying process. Even if you believe that people should be able to choose "rational suicide," you must handle this situation as you would handle any other case in which a client expresses an intent to kill himself. As a social worker, you can be found negligent if you know that a client is suicidal but don't take reasonable steps to protect the client's safety. Depending on the level of risk, these steps can include establishing a no-suicide contract with the client, contacting family members, or having the client hospitalized. In this situation, the comparable step would be to inform the hospice's medical staff, so that the patient will be monitored

The parents of a young child seek help from a social worker because their child has been having difficult speaking clearly. The child has been stuttering, and they're worried that he will never speak fluently and will feel different from others as a result. The social worker is MOST likely to conclude that the child's stuttering is normal childhood speech dysfluency if: Select one: a. the child has no associated emotional or behavioral problems. b. the child is 3 years old. c. the onset of the child's speech difficulties was before age 7. d. the child displays only one type of dysfluency

B: Speech dysfluencies are common in preschool children (ages 2 to 4), and most outgrow them and develop normal speech without intervention

The term "reasonable accommodation" is MOST closely associated with which of the following? Select one: a. Special education. b. Disability laws. c. Affirmative action. d. Patients' rights

B: The Americans with Disabilities Act (ADA, P.L. 101-336), enacted in July 1990, prohibits discrimination against and ensures equal opportunity for persons with disabilities in employment, state and local government services, public accommodations, commercial facilities, and transportation. Under this law, a qualified individual with a disability is any person with a disability who, with or without "reasonable accommodation," can perform the essential functions of the job he or she holds or would like to be hired for. Reasonable accommodation involves making existing workplace facilities readily accessible to and usable by persons with disabilities and or or job restructuring, modified work schedules, acquisition or modification of equipment, modification of tests, training materials, and policies, and other similar accommodations. Workplace discrimination under the ADA is defined as not making reasonable accommodation to the physical or mental limitations of an otherwise qualified person with a disability who is an applicant or employee, unless the employer can show that the accommodation would impose an undue hardship on the operation of his or her business; or denying job opportunities to an applicant or employee who is an otherwise qualified individual with a disability, if this denial is based on the need to make reasonable accommodation to the physical or mental limitations of the applicant or employee

When using the DSM-5, a diagnosis of major neurocognitive disorder requires the presence of a significant decline in: Select one: a. social and occupational functioning from premorbid functioning. b. cognitive functioning that interferes with independence in everyday activities. c. at least three areas of cognitive functioning. d. the ability to perform activities of daily living

B: The DSM-5 distinguishes between major and mild neurocognitive disorders. The diagnosis of a major neurocognitive disorder requires a significant decline from a previous level of cognitive functioning that interferes with independence in everyday activities; while the diagnosis of a mild neurocognitive disorder requires a modest decline from a previous level of cognitive functioning that does not interfere with independence in everyday activities.

As described in the DSM-5, level of severity of alcohol use disorder and other substance use disorders is based on: Select one: a. degree of functional impairment. b. number of symptoms. c. the presence of tolerance and or or withdrawal. d. the degree of distress caused by use of the substance

B: The DSM-5 distinguishes between three levels of severity of the substance use disorders based on the number of symptoms: mild (2 or 3 symptoms); moderate (4 or 5 symptoms); severe (6 or more symptoms).

A hospital social worker is meeting with a Chinese-American man, age 50, and the man's mother, age 80; the mother was hospitalized one week ago. The physician has recommended that the mother be placed in a nursing home. She has become disoriented and frail and her memory is very impaired. The social worker discusses the doctor's recommendation with the man while his mother is also in the room. The man listens politely for a few moments and then says that his mother is fine and can continue living at his home. Which of the following would be the BEST way to interpret this situation? Select one: a. The man is unwilling to accept the severity of his mother's condition. b. The man is not comfortable discussing this issue with the social worker. c. The man doesn't understand the nature of nursing home care. d. The physician's recommendation was premature

B: The question doesn't tell us much about this man, other than his cultural background. This suggests that you, as a test-taker, need to interpret this situation from within a cultural framework. It also means that you have to assume that this man adheres to values that are characteristic of traditional Asian cultures. From this perspective, Answer "B" is correct for two reasons: First, Asian-American individuals may be uncomfortable discussing family issues with an "outsider," such as a therapist, particularly issues concerning their parents or other elders. Second, Asian-American individuals tend to value interdependence and responsibility to the family (as opposed to independence and autonomy) and respect for their elders (as opposed to egalitarian values).

A social worker meets with a same-sex couple who wants help learning how to communicate with each other because they fight too much. In the first interview, the clients stipulate that they don't want to talk about being gay or anything else having to do with sexuality. What is the social worker's BEST course of action? Select one: a. Avoid discussing issues related to sexuality and focus on assessing the clients' communication and interaction patterns by asking them to role-play a typical fight. b. Allow the clients to present the problem as they perceive it, but let them know that she may have ideas to share about the problem once she's heard what they have to say. c. Avoid focusing on the clients' sexuality but explain that it may play a role in their problem even if they are unaware of its effects. d. Interpret the clients' request as an indicator of internalized homophobia and discuss this openly with them to facilitate trust

B: This is the best answer offered. Some gay clients present with concerns that relate in a direct way to their sexual orientation but the majority seek therapy for reasons that have nothing to do with their sexual orientation or are related to it only indirectly. Therefore, the couple's sexuality may have no bearing on the problem. On the other hand, before you formulate a treatment plan, you must understand your clients' problems as fully as possible, and this understanding should incorporate both how the clients initially define their problem and any other relevant issues that you uncover during assessment - e.g., issues that play a role in the problem but that the clients were not aware of when they initially sought therapy

A client begins crying during his first therapy session. What should the social worker do? Select one: a. Reassure the client that he'll feel better now that he's in therapy. b. Sit quietly with the client until he is ready to speak. c. Ask an open-ended question that invites the client to tell her why he is crying. d. Ask the client if she can come sit beside him

B: This question has a "best" answer. When a client cries during a therapy session, you wouldn't want to interrupt his expression, at least not right away. As suggested by "B" more than any of the other responses, you should just allow the client to cry for a little while. The reassurance in "A" would probably come off as ungenuine to the client. Exploring why the client is crying ("C") is appropriate, but first you should let him cry for a few moments and then give him a chance to recover from his strong emotion. Finally, "D," especially in the first session, may make the client uncomfortable and inhibit him from freely expressing his feelings

A client who has been in individual therapy for three months has recently started to make progress toward meeting her goals. The client then terminates treatment without any warning. What is the social worker's BEST response to this? Select one: a. Respect the client's right to terminate treatment when she wants to. b. Attempt to make a final contact with the client by phone or letter. c. Document in the client's case record the reasons and circumstances surrounding the termination. d. Wait for the client to contact you and then warmly encourage her to return to therapy

B: When a client terminates treatment without warning (is a "no-show"), you should attempt to make a final contact with her by phone or letter. The goal of this contact may be to acknowledge her decision to terminate, encourage her to come in for a final session so that you can bring your relationship to an appropriate closure, and or or achieve the purposes of a final session through the phone call or letter. In the phone call or letter, for example, you can review the goals the client has achieved, reaffirm your regard for the client, and inform her of other services available to meet her ongoing needs. You would also document in the client's case record information about her termination ("C"), but the more important issue at the heart of this question is your awareness that you should make an effort to contact a client who is a "no-show" because doing so is in her best interests

A client is a middle-aged man who complains of anxiety. He is feeling overwhelmed by his responsibilities at home and work and as coach of his daughter's softball team. He says things got worse when his mom broke her hip and asked him to help out at her house a few days a week. The client feels like he's spread too thin and letting people down. From the perspective of role theory, this client appears to be experiencing: Select one: a. self-role incongruence. b. role conflict. c. role overload. d. role incapacity

C: A social role problem involves either difficulties in fulfilling social role obligations or conflicts in relationships with family members and or or with people at work (or school) or in the community. One cause of social role problems is role overload which can emerge when a person occupies more roles than he can perform adequately. Because the question emphasizes the client's many social roles and his sense of being overwhelmed and unable to perform all of his roles adequately, role overload is the best answer. Self-role incongruence ("A") is the cause of a social role problem when there is little overlap between the requirements of a role and the individual's personality (e.g., a person finds that his values conflict with the expectations of a role). Role conflict ("B") is the cause of a social role problem when a person is faced with opposing role expectations within one social role (intra-role conflict) or with opposing expectations associated with two different social roles (inter-role conflict). Finally, role incapacity ("D") is the cause of a social role problem when an individual cannot adequately perform a social role. Possible reasons for this include physical or mental illness, substance addiction, intellectual disability, a lack of knowledge or skills, inadequate socialization, or inadequate resources from the environment. In this case, all you really know about this man is that he seems to be having difficulty juggling his multiple social roles.

Which of the following best describes an "ecosystem"? Select one: a. A community and those who live in the community. b. An individual and his social support system. c. An individual, other living systems, and their physical environment. d. An individual and his home and workplace

C: An "ecosystem" is a biological community of interacting organisms and their physical environment and is the unit of study in "ecology." Ecology, in turn, examines the relationship between an organism and its biological and physical environments, including how the organism adapts and functions in these environments. In social work, the "ecosystems perspective" combines ecological concepts with systems theory and proposes that human development should be viewed within a context that includes an individual's relationship with his environment and that each environment is unique. Germain, who applied ecosystems concepts to social work, advocated a "transactional" view of the person-environment relationship which proposes that a person and his environment engage in ongoing circular exchanges in which they reciprocally influence each other over time

A social worker discovers that a client diagnosed with bipolar I disorder has been exhibiting symptoms of the disorder for the past week. What is the social worker's BEST course of action? Select one: a. Have the client hospitalized. b. Verify that the client has been taking her medication. c. Arrange for the client to see her psychiatrist. d. Contact the client's family for collateral information and support

C: Because effective treatments for bipolar I disorder rely primarily on medication (e.g., lithium), you should have the client see her psychiatrist as soon as possible. In particular, patients with bipolar I disorder may not comply with their medication regimen because they feel better and think medication is unnecessary, are unwilling to give up the "highs" of mania, or do not like the drug's side-effects. Answer "B" alludes to this problem, but is incorrect because, as a social worker, you are not qualified to address this problem in a direct way - i.e., you could help the client cope with effects of the medication, but could not advise her in any way about how to use the medication or adjust her dosage. And the fact that the client is "exhibiting symptoms" of bipolar I disorder doesn't necessarily indicate a need for hospitalization ("A"). Hospitalization is called for in cases of severe mania (or severe depression) in which the client poses a danger to self or others.

A client arrives for her first interview while in the midst of a manic episode. The client says that she has decided to get a divorce and asks the social worker to assist her. What should the social worker do FIRST? Select one: a. Confront the client with the fact that she is not stable now and may change her mind about the divorce when the manic episode is over and arrange for her to see her psychiatrist. b. Refer the client to an attorney because a social worker is not qualified to assist a client with divorce proceedings. c. Summarize the client's wishes and arrange for her to undergo a psychiatric medication evaluation. d. Narrow the focus of the interview by asking questions that assist the client to describe her concerns about her marriage.

C: Because the client should probably be taking medication such as lithium (or may have stopped taking medication that was prescribed in the past), you should arrange for her to undergo a psychiatric medication evaluation. Confronting the client ("A") while she's experiencing a manic episode would be futile. Once she's stabilized on medication, she may not even remember this conversation. Answer "B" focuses on the fact that you're not qualified to provide a client with legal advice. The significant issue in this case, however, is that the client is in the midst of a manic episode and, therefore, her judgment - including her judgment about ending her marriage - is almost certainly compromised. Referring her to a divorce attorney would not be appropriate at this time. Finally, attempting to explore the client's concerns about her marriage ("D") is not appropriate while she's in the midst of a manic episode. Instead, the client needs to be stabilized on medication (and, possibly, via hospitalization, if she is dangerous or gravely disabled).

To receive Title IV-B funding for foster care, states are required to provide certain protections to all children in foster care. These protections include all of the following EXCEPT: Select one: a. regular visitation between the child and his biological parents. b. a plan for attaining permanence for the child. c. placement in close proximity to the child's biological parents. d. placement consistent with the child's special needs

C: Foster children are not guaranteed visitation with their biological parents - e.g., in some cases the parents are dangerous and/or there is no possibility of reuniting the family. When visitation is indicated, however, the parents should visit their child in foster care so that the attachment can continue and the parents have an opportunity to practice positive interactions with their child. The other answers describe some of the protections that states must provide to all foster care children in order to receive Title IV-B funding for foster care

During the first interview, a client reports feeling extremely guilty about taking supplies from his office to use at home. After she has acknowledged the client's feelings about this, what should the social worker do FIRST? Select one: a. Identify whether the client's poor judgment is pervasive or related to only this area of functioning by assessing impulse control in other areas. b. Recognize that pervasive feelings of guilt may indicate a depressed mood and ask the client about other symptoms he has been experiencing. c. Invite the client to continue expressing himself and respond to him without judgment. d. Let the client know that people usually feel guilty about stealing in order to normalize his guilt and reduce his distress

C: In the first meeting with a client, you should initially focus on whatever the client considers important to talk about and avoid jumping to conclusions about his problems or concerns. Unless you need specific data, you should ask mostly open-ended questions and focus on giving the client an opportunity to explore and clarify his thoughts and feelings about his behavior in a safe way. You should also focus on conveying acceptance, as this can facilitate the development of trust. While it's possible that the client has pervasive poor judgment or low impulse control or is depressed ("A" and "B"), it's too early in the assessment for you to make either of these inferences. More information is needed beforehand, from both the client and other sources. Although reassuring a client that his feelings are similar to those of other people in similar circumstances can often be useful ("D"), this client probably already knows that most people feel guilty after they steal. This statement also has a bit of a judgmental tone, which you would want to avoid. The correct answer is: Invite the client to continue expressing himself and respond to him without judgment

A social worker is meeting with a 30-year-old woman who reports having difficulty sustaining meaningful relationships. In the first interview, the client expresses a strong desire to change and says she hopes that therapy will enable her to connect better with others. During the next two sessions, the client talks at length about her former relationships and theorizes about why they ended. She tells many detailed stories about old friends and boyfriends. When the social worker tries to redirect the client to an exploration of how she might improve her interpersonal skills, the client resists this and continues telling stories about her past. What strategy should the social worker use to redirect the client's energy toward efforts at therapeutic change? Select one: a. Point out her successes and strengths as she describes her relationships. b. Empathize as she talks about her relationship difficulties. c. Work with her to establish clear short-term objectives with time-frames for completion. d. Confront her resistance to change

C: It's still early in treatment. As this client's therapist, you would want to avoid both framing this problem right away as "resistance to change" and confronting the client too early. For example, rather than being resistant to change, perhaps the client misunderstands the purpose or process of therapy. Establishing clear short-term objectives, with time-frames for completion, could be very effective for mobilizing this client's energy for therapeutic change because perceiving concrete gains (i.e., reaching an objective), even small gains, could motivate her to work toward additional change

The client is a 30-year-old man who was diagnosed with schizophrenia five years ago. The client has experienced several acute psychotic episodes, including full-blown delusions. He is currently in a stable phase of his illness. When developing an outpatient treatment plan for this client, the social worker is MOST likely to emphasize which of the following? Select one: a. Management of medication side-effects. b. Interpreting the delusions to minimize their effects on his functioning. c. Social problem-solving. d. Referral for vocational training

C: Many people with schizophrenia have difficulty solving problems arising from day-to-day living and stressful life events. These deficits can affect their ability to live independently and contribute to disability and poor quality of life. Problem-solving therapy is designed to improve the ability to approach problems in a systematic way, and this therapy can be used in addition to antipsychotic medication and other supportive interventions. The research suggests that, for optimal effectiveness, the remediation of social problem-solving skills with this population should focus on role-playing (nonverbal) interpersonal behaviors, rather than on verbally analyzing an interpersonal problem and clarifying alternative solutions. You are asked what the treatment plan may emphasize; the intervention in "A" is important, but too limited to be the correct answer. "B" is contraindicated for people with schizophrenia because it can lead to confusion and decompensation. In general, insight-oriented psychotherapies have not been found effective for individuals with schizophrenia. And, finally, although the client may benefit from vocational training ("D"), this intervention, like "A," is too limited

A social worker meets with a mother, father, and 6-year-old son. For the past three weeks, the son has been wetting the bed nearly every night. The parents say that they are surprised and concerned by this problem because the boy has been fully toilet trained for several years. They add that they haven't observed other emotional or behavioral changes in their son. What should the social worker do FIRST? Select one: a. Facilitate the parents' understanding of the problem and treatments for the problem through psychoeducation. b. Clarify the problem by identifying its underlying causes through assessing family dynamics and/or referring the boy to a psychologist for psychological testing. c. Clarify the problem by arranging for the boy to be seen by a physician for a medical evaluation to determine if the problem has a physical cause. d. Gather information to complete a family history to determine whether other family members have had this disorder and how it was handled

C: Physiological causes for the boy's symptom, such as a medical condition, must be ruled out before you can identify what other assessments are needed, determine a diagnosis, or decide on appropriate treatments

If left untreated, diabetes mellitus can result in which of the following symptoms? Select one: a. Emotional lability, obesity, memory loss, depression. b. Accelerated heart rate, nervousness, agitation, fatigue, insomnia. c. Increased appetite with weight loss, apathy, confusion, mental dullness. d. Lethargy, slowed heart rate, depression, impaired concentration and memory

C: The pancreas releases insulin, which is involved in the uptake and use of glucose and amino acids, and hypoinsulinism produces diabetes mellitus (excessive blood glucose). Untreated diabetes mellitus may result in increased appetite with weight loss, apathy, confusion, mental dullness, polyuria, polydipsia, and increased susceptibility to infection. The symptoms in "A" are associated with Cushing's disease, which results from hypersecretion of cortisol. The symptoms in "B" (along with a speeded up metabolism, elevated body temperature, heat intolerance, and increased appetite with weight loss) are associated with hyperthyroidism (Grave's disease). And the symptoms in "D" (along with a slowed metabolism, reduced appetite with weight gain, lowered body temperature, and decreased libido) are associated with hypothyroidism

In asthma, the muscles of the walls of the bronchi contract, making it difficult to breathe. This response is triggered by which branch of the nervous system? Select one: a. Autonomic nervous system. b. Sympathetic nervous system. c. Parasympathetic nervous system. d. Central nervous system

C: The parasympathetic nervous system is associated with the stimulation of smooth muscles; in asthma, it causes bronchioles to narrow. Answer "A" (autonomic nervous system) is almost correct, but you are given a more specific answer to select. The autonomic nervous system (ANS) regulates the activities of the visceral muscles and glands and is primarily associated with involuntary movements. It consists of the sympathetic and parasympathetic branches. The sympathetic nervous system ("B") is associated with the inhibition of smooth muscles; it would cause relaxation of the bronchioles. The central nervous system ("D") consists primarily of the brain and spinal cord

The client is a 41-year-old married man who reports feeling tired a lot of the time and having tremendous difficulty concentrating at his job. He no longer enjoys his work and finds that he is easily annoyed by his family and friends. He adds that he thinks he's depressed but doesn't know what to do to "pick himself up." In an effort to understand the client's needs, the social worker assesses him for suicidal risk; gathers information about his health history, medication use, and health and lifestyle behaviors; and performs a mental status exam to assess his psychological status. What should the social worker do NEXT? Select one: a. Use the mental status exam findings to formulate an accurate diagnosis of the client's mental health condition. b. Identify the extent to which medical factors are affecting the client's psychological functioning and vice versa. c. Gather information on the client's cultural background, social relations, and environmental connections. d. Gather information on the client's family by using a genogram to map family pattern

C: The question describes two of the three components of a comprehensive biopsychosocial-cultural assessment - "bio" and "psycho." "C" describes the third component. To complete your assessment of this client, you must gather information about his sociocultural experiences. The diagnosis ("A") should be formulated after you've completed a biopsychosocial-cultural assessment. In addition, the diagnosis should be derived from an evaluation and integration of all of the information you gathered during assessment, not just from the MSE results. Answer "B" is a consideration during the diagnostic process - you need to complete your assessment before formulating a diagnosis. Finally, "D" touches on the need to assess the client's sociocultural experiences, but it's more limited than "C."

A social worker is beginning family therapy with a family of four, including the father, mother, and two children, ages 18 and 19. The mother was recently diagnosed with a serious mental disorder and is taking medication which is helping to control her symptoms. During the first session, the father and children focus on talking about the mother's insurance coverage. What is the social worker's BEST course of action? Select one: a. Point out and address the family's denial. b. Redirect their focus. c. Reframe the family's behavior as concern. d. Allow them to continue talking about insurance coverage until they are ready to talk about the mother's illness

C: The question says that you're meeting for the first time with the family. The family is confronting a serious psychosocial stressor and would benefit from receiving support and guidance from you. "Reframing" is often effective for reassuring clients, especially at the beginning of treatment. Reassurance, in turn, is a form of sustainment that can be very effective for reducing anxiety and offering clients a feeling of hope about their situation. Lessened anxiety and greater hope, in turn, can help free up clients' emotional and mental energies so that they can begin to engage in problem-solving activities

A client has been in therapy since the break-up of his marriage several months ago. He arrives for a session looking disheveled. After a few minutes, he tells the social worker that he can't eat or sleep. As he speaks, his voice quivers and he appears to be in significant emotional distress. In terms of assessing these changes in the client's functioning, what should the social worker do FIRST? Select one: a. Assess for major depressive disorder and refer the client for physical evaluations. b. Evaluate the degree of disequilibrium and immobility to determine whether the client is in crisis. c. Determine threat to life by asking direct questions. d. Identify stressful events that have occurred since the last session and explore the client's perceptions of them

C: This client appears to be in acute crisis, and "C" is the only answer that immediately addresses the possibility that he may pose a danger to himself (or perhaps to others) at this time. The first thing you should do is conduct a suicide assessment, including asking the client directly whether he has thought about harming himself. It's a good idea to evaluate whether the client has developed major depression, as "A" suggests, but not as a first step. "B" isn't the best choice because, in this situation, you need to focus directly on whether the client is suicidal. Finally, "D" will be useful for understanding the client's altered condition and what can be done to help him, but, at this time, it's more important to conduct a suicide assessment

A social worker begins working with an American Indian couple. The couple has two young children, and the family has basic needs related to poverty. In addition, the social worker determines that the father drinks alcohol daily. She attempts to engage the couple in a discussion about the husband's alcohol use but the couple persistently avoids talking about this issue. The social worker helps the couple meet some of their basic needs for food and adequate shelter. Which of the following is MOST likely to be true about this situation? Select one: a. The couple is in denial about the alcohol problem. b. The couple's basic needs are urgent and they will be willing to address the alcohol use once their basic needs are met. c. The couple does not consider alcohol use to be a problem and just wants help meeting their basic needs. d. The couple is embarrassed by the husband's alcohol use

C: This is a difficult question because it requires you to be very familiar with the literature on counseling American Indian clients. So let's take a look at why "C" is the best answer: First, substance use disorders are a significant problem faced by American Indians, and a number of explanations have been proposed to explain this, especially the prevalence alcohol use disorders (Sue and Sue, 1999): One explanation is that refusing an offered drink is viewed an act of individual autonomy that disrupts group harmony; another is that alcohol use allows American Indians to release feelings that are normally kept under control (e.g., frustration and boredom). Others (e.g., Manson, Tatum, and Dinges, 1982) have pointed out that drinking is an accepted practice among many American Indians and is encouraged among family members. This information suggests, as answer "C" says, that these clients may not consider the husband's alcohol use to be a problem. Second, Sue and Sue (1999) suggest that when working with American Indian clients, a therapist may need to address basic needs first, including problems stemming from poverty such as inadequate food, shelter, child care, or employment. One reason for this is that many traditional American Indians are more oriented to the present than the future. If this is true about these clients, then they are likely to prefer interventions that work in the here and now. (See, Manson, S. M., Tatum, E., and Dinges, N. G. [1982]. Prevention research among American Indian and Alaska Native communities: Charting future courses for theory and practice in mental health. In S. M. Manson, [Ed.], "New directions in prevention among American Indian and Alaska Native communities." Portland, OR: Oregon Health Sciences University.)

A social worker begins working in individual therapy with a woman who has been physically abused by her husband on multiple occasions. The woman acknowledges the abuse but doesn't want to leave her husband. The husband has been ordered into a batterer intervention program by the court, and the woman believes that things will change now that he's finally getting help. Research on batterer intervention programs and similar court-ordered treatments for men who batter their intimate partners has found which of the following to be true? Select one: a. They are effective for terminating physical violence. b. They are effective for reducing both physical violence and more subtle forms of abuse. c. They can be effective for reducing physical violence but do not usually reduce more subtle forms of abuse. d. They have no effects at all because offenders rarely complete the program

C: This question is difficult, in part, because it requires you to be familiar with the research on batterer intervention programs (BIPs) (a.k.a. spouse abuse abatement programs, or SAAPs) and, in part, because the results of this research have been inconsistent, mostly because of differences in evaluation methods. According to a report published by the U.S. Department of Justice (Jackson et al., 2003), some available data suggest that court-ordered treatment for batterers correlates with a reduction in physical violence, but neither terminates violence in many cases nor reduces the more subtle forms of abuse. (Whether it's the treatment itself or individual motivation brought on by legal intervention that causes the reduction of violence is unclear, however.) Other studies have found that men arrested for intimate partner violence who receive treatment resume violent behaviors as often as do men arrested and not referred to treatment, and that there is no significant difference between men who complete batterer's treatment and men who drop out of the program

A client reports that her elderly father has seemed rather confused lately. A physician recently evaluated the man, and all neuropsychiatric tests for a neurocognitive disorder have been negative. The physician found no major medical problems. The client says that she has decided to move her father (whose wife died 10 years ago) to an independent living facility. What is the social worker's BEST course of action? Select one: a. Review the doctor's findings with the client and discourage her from taking rash action concerning her father. b. Find out why, given the medical findings, the client wants to place her father. c. Seek permission from the client to conduct an in-home assessment of her father. d. Begin helping the client locate an acceptable independent living facility

C: You (the test-taker) don't know where the father is living (does he live alone?), and the question suggests that, so far, you have met with only the daughter; in other words, you haven't had an opportunity to observe or evaluate the client's father. "C" is the best answer offered because you need more information about the father's functioning before making any placement recommendations to the daughter or even forming an opinion about whether relocation is indicated

An adolescent client has been experiencing frequent tics for many years and was recently diagnosed with Tourette's disorder. In providing this client with psychoeducation about his condition, a social worker would be correct if she told him all of the following EXCEPT: Select one: a. many adults with tic disorders experience diminished symptoms. b. treatment may involve administration of an antipsychotic medication. c. the symptoms cannot be voluntarily suppressed. d. the symptoms may wax and wane in frequency

C: You needed to choose the answer that states something that is false about Tourette's disorder. Tourette's disorder is characterized by the presence of at least one vocal tic and multiple motor tics that may appear simultaneously or at different times, that may wax and wane in frequency but have persisted for more than one year, and that began prior to 18 years of age. According to the DSM-5, "tics are generally experienced as involuntary but can be voluntarily suppressed for varying lengths of time" (p. 82); therefore, "C" is a false statement. Answer "A" is true. According to the DSM-5, the onset of tics is usually between ages 4 and 6 years; peak severity occurs between ages 10 and 12 years, with a decline in severity during adolescence; and many adults with tic disorders experience diminished symptoms (a small percentage of individuals, however, have persistently severe or worsening symptoms in adulthood). Answer "B" is also true - antipsychotic drugs have been found effective in about 80 percent of cases. As noted in the explanation for "C," "D" is also true

A social worker is conducting an interview with a client who expresses a tremendous amount of anger about something that happened during the week at his job. The social worker realizes that she needs to help the client regain control so that his anger doesn't escalate to a point where he becomes dangerous. To achieve that goal, the social worker may do all of the following EXCEPT: Select one: a. admit her error and apologize if she says something that causes the client to become more angry. b. let the client know exactly what she needs from him right now. c. give the client specific advice on how to manage his angry feelings. d. give the client supportive feedback to mitigate his angry feelings

C: You should avoid giving advice to an angry or otherwise potentially violent client. Letting an angry client know what you need from him so that you can help ("B") may serve to lessen his anxiety by providing structure. For example, "You seem very angry. I'd like you to sit down and tell me exactly what has happened and we'll figure out how it led to your anger." And supportive feedback ("D") can mitigate the client's anger, fears, hostility, or frustration.

A client, age 79, has come in alone for treatment. He is exhibiting some signs of depression. To clarify the nature of the problem in this case, the social worker should FIRST: Select one: a. ask the client how he spends his time. b. explore ways for the client to expand his support system. c. evaluate what losses the client has experienced in this stage of his life. d. refer the client to a neurologist for testing

D: "D" is overly specific but it is still the best choice offered (be prepared to see "imperfect" answers on the exam, too). That is, you would probably first refer the man to his regular physician for a general health screening, and the physician might then recommend that the man see a neurologist. We still choose "D," however, because, before formulating the problem or planning treatment, you must determine an accurate clinical diagnosis (if any) for this man. This requires confirming or ruling out any effects of a medical problem or substance (including medication) on the client's mood and cognitive functioning. In addition, the question doesn't describe the nature of the man's depressive symptoms, but, in depressed older adults, memory loss, distractibility, disorientation, and other cognitive symptoms may be present. If the man has prominent cognitive symptoms, neurological testing could be helpful for differentiating a depressive disorder from mild neurocognitive disorder

A 24-year-old client is referred by his family physician for help in dealing with persistent feelings of sadness and apathy that emerged when he moved from his small hometown to the city. The client says that he's had depression before but it's worse than ever now that he's away from his family and friends for the first time. If the social worker were to use the cognitive theory described by Aaron Beck, she would consider the client's depression to be attributable to: Select one: a. uncontrollable events. b. object loss. c. inadequate reinforcements. d. automatic thought

D: Aaron Beck's cognitive theory has been very influential in therapy with depressed individuals. Beck believes that depression is the result of negative automatic thoughts that are triggered by environmental stress. As a cognitive therapist, he focuses primarily on the role of cognitive "errors" in depression and other disorders. One of the first targets of his treatment are the "automatic thoughts" that underlie a client's maladaptive behaviors and emotions. Answer "A" sounds more like Seligman's theory of learned helplessness; "B" is based on an object relations model; and "C" is associated with a behavioral approach.

Cognitive dissonance theory would predict that which of the following individuals would feel best about participating in therapy? Select one: a. A wealthy individual who has no trouble paying for therapy. b. An individual whose insurance company pays 75% of his or her therapy fee. c. An individual who receives therapy for free. d. An individual who has to work overtime to pay for his or her therapy

D: According to cognitive dissonance theory, an individual is motivated to reduce the dissonance (a negative, aversive state) that results when his or her cognitions are in conflict. Thus, an individual who has the conflicting cognitions "I am working overtime to pay for therapy" and "I am not benefiting from therapy" would experience a state of dissonance: this individual, more so than the others in the answers, would be motivated to believe that he or she is benefiting from therapy

Which of the following statements about the need to obtain valid consent before releasing confidential information about a client is MOST true? Select one: a. A social worker must always obtain valid consent from a client or a client's legal representative before releasing confidential information. b. Before consulting with a colleague about a client's case, a social worker must always obtain valid consent from the client. c. When a social worker wishes to release confidential information to a client's family members, he or she may do so without obtaining valid consent from the client. d. A social worker must obtain valid consent from the client some of the time before releasing confidential information

D: Although it might sound wrong because of the words "some of the time," answer "D" is a true statement. The general rule is to avoid releasing confidential client information to third parties without permission (valid consent) from the client or his or her legal representative. Such "permission" should consist of a signed release of confidential information (a "release"). However, there are situations in which a social worker is permitted or required to disclose confidential information without such permission. Specifically, NASW's Code of Ethics says that the assumption that you will keep information confidential does not apply when you need to disclose certain information to prevent serious, foreseeable, and imminent harm to a client or another identifiable person or when laws or regulations require you to disclose certain information without a client's consent. You don't need to have a client's permission to discuss his or her case with a consultant ("B") as long as you do not disclose information that would reveal the client's identity to the consultant. And, you do usually need to have a client's valid consent before sharing confidential information with his or her family members ("C"). One exception would be an emergency in which the client's safety is threatened (e.g., the client is suicidal) and you believe that notifying members of the client's support system is necessary to protect the client, yet the client refuses (or is unable) to give you permission to speak to his or her family members

A group of social workers involved in an episode of macro change have finished planning the intervention, including selecting an approach to change and establishing the intervention's goal and objectives. In the next phase of their work and prior to actually implementing their intervention, these social workers are likely to do all of the following EXCEPT: Select one: a. select a lead person or coordinator. b. acquire and organize resources needed to launch the intervention. c. hire additional personnel. d. develop a working hypothesis of intervention

D: Change agents must have a working hypothesis of intervention before planning the intervention. The major steps involved in an episode of macro change include the following: (a) Analyze the problem, population, and arena (e.g., interview those affected by the problem, create a condition statement, explore literature related to the problem and population, collect relevant data, prepare collected data for presentation to decision-makers). (b) Develop a working hypothesis of etiology about the problem. (c) Develop a working hypothesis of intervention based on relevant findings from the earlier steps and the working hypothesis of etiology. (d) Select an approach to change (policy, program, project, practice, or personnel). (e) Build support for the change (e.g., create coalitions, assess each system's readiness to support the change, examine resources available and resources needed). (f) Estimate the probability of success (i.e., weigh the relative strength of supporting and opposing forces). (g) Decide whether to pursue the change effort. (h) Select strategies and related tactics to get the change approved by relevant decision-makers and others. Available strategies include collaboration, campaign, and contest. (i) Plan the intervention (e.g., establish its goal, develop outcome objectives and process objectives, list activities for process objectives). (j) Prepare to implement the intervention (e.g., select a lead person or coordinator, address logistical considerations such as facilities, equipment, personnel and other resources). (k) Monitor the intervention. (l) Evaluate the effectiveness of the intervention

An adult client who is distraught over the death of her mother has scheduled an appointment with a social worker. The social worker recently lost his own mother and he starts crying with the client during the first interview. From an ethical standpoint, what should the social worker do? Select one: a. Seek consultation. b. Continue working with client as though nothing has happened. c. Work with the client on grief issues but monitor himself closely. d. Refer the client to a different therapist

D: Especially because this is a new client and there is no therapeutic relationship yet, it would be best to refer the client to another therapist for grief work. Section 4.05 of NASW's Code of Ethics states that social workers should not allow their own personal problems, psychosocial distress, legal problems, substance abuse, or mental health difficulties to interfere with their professional judgment and performance or to jeopardize the best interests of people for whom they have a professional responsibility. It also says that social workers whose personal problems, psychosocial distress, legal problems, substance abuse, or mental health difficulties interfere with their professional judgment and performance should immediately seek consultation and take appropriate remedial action by seeking professional help, making adjustments in workload, terminating practice, or taking any other steps necessary to protect clients and others. Therefore, as "A" suggests, you should seek consultation, as well. First, however, you need to meet this client's needs in an appropriate manner by referring her to a therapist who is able to help her at this time

A client reports feeling sadness and loneliness after the death of his mother several months ago. While exploring these feelings with the social worker, the client suddenly withdraws from the conversation, saying he doesn't want to talk about his feelings anymore. Later in the session, the client blurts out that he also felt relieved when his mother died. What should the social worker do FIRST in responding to that statement? Select one: a. Explore why the client felt relieved. b. Interpret the statement as a way of helping the client understand how he really feels. c. Use the technique of mirroring to reduce the client's defensiveness. d. Use the technique of universalization to let the client know that this feeling is norma

D: For this question, you need to be familiar with how people may express and experience grief. Grief may be expressed emotionally in many different ways, including sadness, anger, self-reproach, anxiety, loneliness, helplessness, numbness, and, sometimes, relief or a sense of freedom. This client's behavior in the session suggests that he's uncomfortable with his sense of relief about his mother's death. While you may end up also exploring this feeling with the client, the first thing you would do is let the client know that this feeling is normal (that other people feel it, too). When applying the technique of universalization (a form of reassurance), you make statements that explain to a client that his thoughts, feelings, or behavior are similar to those of other people in similar circumstances. The purpose is to counteract the client's perception that his feelings or behaviors are strange or abnormal

The client is a middle-aged man who sought therapy two days after learning that he has cancer and his prognosis is poor. His doctors have told him that his life expectancy is about six months. The client is in the first stage of Kubler-Ross's (1969) model of death and dying, which describes the stages that people go through when facing their own death. Given this information, the social worker would MOST expect the client to say which of the following when discussing his diagnosis and prognosis with her? Select one: a. "I don't care; I hate my life anyway." b. "Why is this happening to me? I don't deserve this." c. "This must be a punishment for something I did." d. "The doctor must be wrong.

D: In sequence, the stages of Kubler-Ross's model are denial, anger, bargaining, depression, and acceptance. The statement in "D" is the best example of denial. The statements in "A" and "C" are more characteristic of the fourth stage (depression); and the statement in "B" is more characteristic of the second stage (anger)

The client is a recently divorced woman who has sole custody of her 14-year-old daughter. The client is distraught over her daughter's behavior and still depressed about her divorce. She tells the social worker that her daughter has been going out too much lately and becoming secretive about her behavior outside the home. The client is afraid that her daughter is sexually active. From the facts given so far, what is the MOST likely explanation for this situation? Select one: a. Multigenerational transmission process. b. Disengaged family structure. c. Enmeshed family structure. d. Developmental crisis in the family life cycle

D: It was important to base your answer on only the information you were given in the question. You don't have the information needed to support the explanation in "A," "B," or "C." In contrast, you do have sufficient information to infer that the client's difficulties may be due to a developmental crisis in the family life cycle: The mother is trying to adapt to a new lifestyle after her divorce, and the daughter is at the beginning of her teenage years and seems to be expressing her need for independence in a rebellious way.

After nine months with weekly sessions, a client in psychotherapy has made no significant progress. She and the social worker have reviewed and revised the treatment plan several times. To behave in an ethical way, what should the social worker do? Select one: a. Arrange to see the client more frequently for a specified length of time. b. Seek supervision or consultation. c. Ask the client what she would like to do. d. Discuss with the client the termination of his services to her and referral to other helping sources

D: Nine months with no significant progress is long enough for you to conclude that this client is not benefitting from your services (especially since you've already tried revising the treatment plan several times). Therefore, to behave in an ethical manner, you should now discuss the termination of your services with this client; and because termination is occurring before the treatment process is completed, you should also offer appropriate referrals to other helping sources. Although you and the client will discuss the termination, you should not leave it up to the client to decide what should be done ("C"). You have an ethical responsibility to be more proactive than that in this situation.

The MMPI-2 can be used to measure which of the following? Select one: a. Intelligence. b. Premorbid functioning. c. Family dynamics. d. Characteristic behavio

D: The MMPI-2 is one of the most widely used self-report inventories for the assessment of personality. Its clinical and validity scales provide information about an examinee's characteristic traits and behaviors, including symptoms, major needs, perceptions of the environment, reactions to stress, self-concept, sexual identification, emotional control, interpersonal relationships, and psychological resources.

A child acquires the ability to think in logical, cause-and-effect terms, learns to solve the problems of conservation, and can adopt other people's perspectives. According to Piaget's approach to intellectual functioning, the child is in which phase of cognitive development? Select one: a. Formal operations. b. Preoperational. c. Sensorimotor. d. Concrete operations

D: The basic premise of Piaget's "constructivism" is that people actively construct their knowledge through interactions with the environment. Piaget proposed that cognitive development follows a predetermined sequence of four stages - sensorimotor, preoperational, concrete operational, formal operations - that is related to biological maturation. The concrete operational stage (7-11 years) is marked by the emergence of logical (cause-and-effect) thinking, including reversibility and the ability to serialize and sequence. As a result, individuals are able to classify in more sophisticated ways, understand part-whole relationships in relational terms (e.g., bigger, lighter), and conserve number, length, volume, and weight. Reversibility and decentration underlie the ability to conserve. A child in this stage is also able to adopt other people's points of view. During the formal operational stage (11 years through the end of adolescence), the individual can think abstractly and relativistically, engage in hypothetico-deductive reasoning, and carry out systematic tests to prove and disprove alternative explanations for observed events. Egocentrism (self-consciousness, self-criticism, and self-admiration) often re-emerges in this phase. The preoperational stage (2-7 years) is marked by the appearance of symbolic functions, which are associated with the use of language. Individuals demonstrate intuitive thinking, engage in symbolic play, can solve problems mentally, and can imitate behaviors that are no longer visible to them. On the other hand, cognitive operations in this stage continue to be limited by several factors, including egocentrism, animism, irreversibility, and centration; i.e., children in this stage are unable to understand that actions can be reversed and they focus on the features of objects that are most noticeable. Irreversibility and centration underlie the preoperational child's inability to "conserve," or recognize that changing one physical dimension of an object doesn't change its other physical dimensions. Finally, during the sensorimotor stage (birth-2 years), the child learns about objects through the sensory information they provide (how they look, feel, and taste) and the actions that can be performed on them (sucking, grasping, hitting, etc.). By the age of 2, the child achieves object permanence (or the "object concept"), which allows him or her to recognize that objects continue to exist when they are out of sight

A social worker receives a phone call from a psychologist who has just started providing therapy to a woman who is a former client of the social worker's. The psychologist asks the social worker to give her a brief synopsis of his work with this client. The psychologist and social worker are good friends. What should the social worker do? Select one: a. Give the psychologist only objective information about the client's therapy. b. Acknowledge that he worked with the client but withhold confidential information until he has a release of information form signed by the client. c. Give the psychologist whatever information she asks for because doing so is the best interests of the client. d. Not even admit that he worked with the client until he has a release of information form signed by the client

D: The fact that the psychologist is a friend doesn't relieve the social worker of the obligation to have permission ("valid consent") from the client before sharing any confidential information or even acknowledging that he ever saw the client for therapy. Confidentiality refers to the obligation of a social worker not to disclose any information about a client obtained during the course of a professional relationship; the only exceptions to this are when the client (or a legally-authorized person) has consented to such disclosure; in consultations, lectures, writings, and similar circumstances if the client's identity is adequately disguised; and when the Code of Ethics and or the law allows or requires a social worker to breach confidentiality (e.g., to report child abuse).

A social worker is providing case management services to a client. She and the client have completed a careful assessment, established the goals that will be pursued, developed a service plan, and determined what resources are most likely to be helpful for the client. The social worker is now ready to connect the client to these resources. In using connecting activities with this client, what is the social worker's ultimate goal? Select one: a. For the client to obtain all of the resources he needs. b. For the client to report a satisfactory experience with the other providers. c. For the client to make these appointments on his own and maintain appropriate contact with these providers. d. For the client to take charge of getting and using the resources he needs

D: The question asks about your "ultimate" goal. Effective case management should result in not only the resolution of the problems identified during assessment, but also in an increase in the client's own ability get and use help effectively - i.e., key signs that disengagement (termination of case management services) is appropriate are the achievement of goals, the ability of the client to help himself more effectively (increased capacity to meet his own needs and increased independence), and the client's appropriate use of help from a helping network

Two social workers are having lunch at a busy restaurant when one of the social worker's clients approaches him to say hello. What should the social worker do? Select one: a. Pretend he doesn't know the client. b. Introduce the client as his friend to the other social worker. c. Pull the client aside and explain confidentiality. d. Tell the client he'll call her later

D: This is the best answer from among the four. By calling the client later, the social worker can explain confidentiality and how it impacted his response to her in the restaurant, as well as avoid breaching confidentiality by having this discussion while at the restaurant. Answers "A" and "B" would confuse the client and potentially damage the therapeutic relationship. "C" is incorrect because a busy restaurant is not the setting to explain confidentiality

A social worker has been working in individual therapy with an adult client for five months, and the presenting problem is in remission. The client tells the social worker that he wants to terminate therapy; the social worker believes that the client would benefit from additional therapy to consolidate the gains he has made. What should the social worker do? Select one: a. Discuss with the client why he wants to terminate and why she thinks he shouldn't. b. Seek case consultation. c. Agree to terminate because the client feels ready to leave therapy. d. Explain to the client why she feels he shouldn't terminate but allow him to do so if he still wants to after that

D: This question illustrates why you must read every answer to a test question before choosing one. Answer "A" is fine, but "D" is better. It's in the client's best interests for you to tell him why you believe he shouldn't terminate yet so that he can make an informed decision. If he continues to want to terminate after that, then you must respect his right to do so. While seeking consultation ("B") is never a bad thing to do, it doesn't seem necessary in this common clinical situation

A client reports experiencing fatigue and excessive nervousness. She says she has a history of periodic depression but has never felt so nervous or anxious before. She was treated for depression two years ago and says that her mood has improved since then. Now, she's just a "nervous wreck" and feels like she can never calm down. What should the social worker do FIRST in assessing this client? Select one: a. Collect a detailed history of the onset, development, frequency, and nature of the client's anxiety symptoms. b. Ask the client about other symptoms she's experiencing to determine whether her depression has returned. c. Ask the client about recent events, stressors, and transitions in her life and explore her responses to them. d. Collect a medical history, including a review of medications, and refer the client for a physical exam

D: This question is difficult because a social worker would do a variety of different things when assessing a client who reports symptoms of anxiety. In fact, three of the answers describe steps that are usually taken in the clinical assessment of anxiety, and "B" would be appropriate, as well, since this client has a history of depression. "D" is the best answer to a question like this one, however, because ruling out the possibility that a medical condition or substance or medication use accounts for a client's symptoms is always a top priority

A young boy expresses a metaphor during a play therapy session. How would a play therapist respond to this? Select one: a. Encourage the child to express himself more directly. b. Restate the metaphor and assess the child's reaction to this. c. Ask the child to describe the meaning of his metaphor. d. Stay with the metaphor

D: When a child in play therapy expresses a metaphor or other symbol in his play, the therapist will usually just stay with it, without forcing the child to explain the symbol or express himself in a more straightforward way. In play therapy, play and play materials are used to encourage a child's expressive behavior, and a play therapist generally avoids directing the child's verbal expressions or play activities (unless the child engages in dangerous play behavior)

A consultant's functions in client-centered case consultation include all of the following EXCEPT: Select one: a. collecting information. b. giving advice. c. problem-solving. d. mandating solutions

D: When offering client-centered case consultation, a consultant works with a social worker (or other consultee) to develop a plan that will enable the social worker to work more effectively with a specific client. The consultant, who is considered an expert in the problem area, gathers information about the client and then makes recommendations to the social worker about the best course of action. The consultant is not an administrator, however. In fact, the consultant has no administrative (or other) authority over the consultee, and the consultee is free to reject his or her advice

A client, age 31, tested positive for HIV two months ago, and, during a therapy session, he reveals to the social worker that he has not told his girlfriend about his condition and sometimes has unprotected sex with her. In this situation, the social worker: Select one: a. must contact the client's girlfriend to inform her that the client is exposing her to HIV. b. must contact the client's girlfriend to inform her that she may have been exposed to HIV but must not reveal the client's identity. c. must contact the client's girlfriend to inform her that she may have been exposed to HIV but only if the client refuses to do so himself. d. should discuss her legal obligations with a knowledgeable colleague or an attorney

D: Whether you have a duty to protect (a.k.a. duty to warn) in this situation is ambiguous, and consequently, your best course of action would be to discuss the situation with a colleague or attorney to determine how you should proceed. (Note that you would also want to inform the client of his duty to inform his girlfriend of his serostatus.)

A social worker is working in individual therapy with a 30-year-old client who is having difficulty making decisions. The client is trying to decide whether to move back into his parents' house so that he can save money to return to college. The social worker and client discuss this decision, including using a decision matrix to consider the pros and cons of each option. The client arrives at a decision that the social worker disagrees with. What should the social worker do? Select one: a. Help the client express the basis for his decision and provide him with more information to facilitate the best decision. b. Review the decision matrix with the client. c. Support the client's willingness to make a decision, but make it clear that she has reservations about his choice and identify her reservations. d. Help the client identify what steps he needs to take to carry out his decision

D: Your job as a social worker is to facilitate problem solving and decision making by your clients - you don't have a right or responsibility to make decisions for them. The question indicates that you've already helped the client consider his alternatives and make a decision, including through using a decision matrix to evaluate the costs and benefits of each option. Therefore, at this time, you need to accept the client's decision, whether you agree with it or not. Although you don't know that this client needs help with carrying out his decision, "D" is the only answer that indicates you would respect his right to make his own decision

The client is a 9-year-old girl who is brought in by her mother. The girl's grades have dropped and she has been spending a lot of time by herself in her bedroom. When the mother tries to engage the girl in a conversation about the changes in her behavior, the girl becomes upset and refuses to talk. The social worker suspects that the girl may be depressed and assesses her for the core symptoms of a major depressive episode. When doing so, the social worker keeps in mind that certain symptoms are common in children with major depressive disorder. These symptoms include: Select one: a. psychomotor retardation, social withdrawal, distractibility. b. somatic complaints, irritability, social withdrawal. c. somatic complaints, sleep disturbances, irritability. d. irritability, social withdrawal, cognitive symptoms

The symptoms of major depressive disorder vary somewhat with age. Somatic complaints, irritability, and social withdrawal are common in children; while aggressiveness and destructiveness sometimes occur in preadolescents (especially boys). In older adults, memory loss, distractibility, disorientation, and other cognitive symptoms may be present, making it difficult to distinguish depression (pseudodementia) from mild neurocognitive disorder

As defined by Freud, defense mechanisms such as regression, repression, and projection can be MOST accurately described as: Select one: a. unconscious mechanisms that help the individual resolve conflicts. b. mechanisms used by the ego to reduce feelings of anxiety. c. mechanisms used by the id to protect itself from its own threatening impulses. d. mechanisms used by the superego to keep the id from expressing its undesirable impulses

This question simply asks you to identify what defense mechanisms do. It may have been difficult to do this, however, because the answers are similar to one another. Defense mechanisms operate unconsciously and are used by the ego to prevent conscious awareness of anxiety-producing impulses, thoughts, desires, etc. Although they do operate unconsciously, defense mechanisms do not help individuals resolve conflicts ("A"); instead, they keep conflicts out of conscious awareness. Defense mechanisms are used by the ego, not the id, and they protect the person, not the person's id, from threatening impulses ("C"); and the ego, not the superego, uses defense mechanisms ("D")


Kaugnay na mga set ng pag-aaral

Tort 4: Negligence: causation and remoteness of damage

View Set

Microbial communities chapter 19a

View Set

Царство грибов(9-6)

View Set

You shall pass! Microbiology final Chapter 19, Chapter 26 Microbial Diseases of the urinary and reproductive systems, Unit 13: Microbial Diseases; Lesson 5: Microbial Diseases of the Digestive System, Microbiology, Exam 4...

View Set

Microeconomics Chapter 7, 14, 22, 24 James Zipperer

View Set